LSAT and Law School Admissions Forum

Get expert LSAT preparation and law school admissions advice from PowerScore Test Preparation.

User avatar
 Dave Killoran
PowerScore Staff
  • PowerScore Staff
  • Posts: 5852
  • Joined: Mar 25, 2011
|
#90419
Complete Question Explanation
(The complete setup for this game can be found here: lsat/viewtopic.php?t=980)

The correct answer choice is (C)

If G does not sit in row 1, from the first rule G must sit in row 2 and H must sit in row 3. I must then occupy a window seat in row 2 or 3:

G3-Q18-d1.png

This information eliminates answer choices (A) and (D).

The only remaining aisle seat is in row 1, and if M were to occupy that seat, then K would have a window seat, which would lead to a violation of the fourth rule. Thus, M cannot sit in the row 1 aisle seat, which eliminates answer choice (E).

The remaining two answer choices—answer choices (B) and (C)—address row 2, of which the window seat is the only remaining seat available. If K sits in the row 2 window seat, then from the fourth rule M would have to sit in the row 3 window seat, forcing I to sit in row 1. But, this causes a violation of the condition in the question stem, and thus K cannot sit in row 2. This information eliminates answer choice (B), and thus answer choice (C) is correct.

The following hypothetical shows that answer choice (C) is possible:

G3-Q18-d2.png
You do not have the required permissions to view the files attached to this post.
 bethavedon
  • Posts: 7
  • Joined: Jun 20, 2016
|
#28811
Could you help explain why answer choice (E) couldn't also be a could be true here?

If neither G nor I are in row 1, then H must be Row 3 Aisle and G must be Row 2 Aisle. So when I play out answer choice (E) "Moore occupies an aisle seat", I get:

Row 3 L H
Row 2 I G
Row 1 K M

(with the left side being window and the right side as aisle) Which looks okay to me - could you help me figure out what I'm missing?

Thanks,

Beth
User avatar
 Jonathan Evans
PowerScore Staff
  • PowerScore Staff
  • Posts: 726
  • Joined: Jun 09, 2016
|
#29005
Hi, Beth,

With a template such as the following:

_ _ _ w
_ _ _ a
1 2 3

and the condition that neither G nor I is in row 1, you get the following possibilities:

K I M w
L H H a
1 2 3

or

M/L _ _ w (with I and either M or L in the 2nd and 3rd window seats)
K G H a
1 2 3

Therefore E is impossible. Moore must be in a window seat. On this game, pay particularly close attention to the sufficient conditions of the conditional statements. The variable that controls most of the outcomes is K. Proceed on this question to determine first that the GH block (acting as a puzzle piece more or less) has to cover the 2nd and 3rd aisle seats. Then consider that the only possible places for K are in row 1 (either window or aisle). In row 2, K would violate the KG :arrow: IM clue. In row 3, K would violate the Kw :arrow: M3 rule.

Get the most out of your LSAT Prep Plus subscription.

Analyze and track your performance with our Testing and Analytics Package.